[University Special Relativity] Lorentz Transformation and Boosts

In summary, the person is trying to find a solution to a Lorentz transformation that involves two boosts of speed, and they are supposed to do it in a particular order. They should first find the boost in the ##x##-direction with a speed of ##v##, and then the boost in the ##y'##-direction with a speed of ##w##. Finally, they are supposed to multiply the two matrices in the opposite order. Is this correct?
  • #1
Athenian
143
33
Homework Statement
Find the matrix for the Lorentz transformation consisting of a boost of speed ##v## in the ##x##-direction followed by a boost of speed ##w## in the ##y'## direction. Show that the boosts performed in the reverse order would give a different transformation.
Relevant Equations
When it comes to Lorentz transformations, I feel like there are countless equations available for it. However, for this particular problem, I used the following equation for the Lorentz transformation and boost.

Lorentz Transformation:
$$S(P, P) = \eta_{\alpha \beta} x^{\alpha} x^{\beta} = x^{\alpha} \eta_{\alpha \beta} x^{\beta} = -c^2 t^2 + x^2 + y^2 + z^2$$

Boost:
##ct' = Act + Bx## ##\Rightarrow## ##(ct') = (A, B, 0, 0) (ct)##
## x' = Cct + Dx## ##\Rightarrow## ##(x') = (C, D, 0, 0) (x)##
##y' = y## ##\Rightarrow## ##(y') = (0, 0, 1, 0) (y)##
##z' = z## ##\Rightarrow## ##(z') = (0, 0, 0, 1) (z)##

Where A is ultimately equal to 1, B = 0, C = 0, and D =1.
*Please note that the above equation for boost is supposed to be a matrix. However, typing it out on the forum didn't work very well for me. Perhaps there's a way to type out a matrix here that I don't know yet.
Unfortunately, I am not entirely confident of the above equations being able to do the trick and ultimately solve for the question. However, my guess is that using the equation written above for "boost", I could perhaps use ##v## and insert it into the ##x##-direction part of the matrix (somehow) and do the same for the speed ##w## in the ##y'##-direction which is on the other (left) side of the matrix equation for boost.

Furthermore, when it comes to Lorentz transformations, I believe the question would involve the Lorentz transformation identity ##I## during the calculation process since "something times its inverse is identity ##I##" (e.g. ##L \cdot L^{-1} = I##).

Other than these small ideas, I have a difficult time knowing how to proceed with solving the question. Does anyone in the community perhaps have any helpful suggestions to help me move forward in solving the problem? Any help would be sincerely appreciated. Thank you very much for reading through this!
 
Physics news on Phys.org
  • #3
Thank you very much for sharing the thread, mitochan!

First off, my apologies for replying this late despite your kind assistance. I had a tough math exam a day and a half ago and I have also taken the liberty to study more in-depth your thread as well as other online sources regarding (Lorentz) boosts.

With that in mind, here are a couple of things I learned (and understood) through my research - which, unfortunately, doesn't amount to as much as I would like.

To begin with, a Lorentz boost (a Lorentz transformation which does not involve rotation) in the ##x## direction would look like the following:
$$\begin{bmatrix} \gamma & -\beta \gamma & 0 & 0 \\ -\beta \gamma & \gamma & 0 & 0 \\ 0 & 0 & 1 & 0 \\ 0 &0 & 0 & 1 \end{bmatrix}$$
The coordinates are written as ##(t, x, y, z)##.

Please note that ##\beta = \frac{v}{c}## and ##\gamma = \frac{1}{\sqrt{1- \frac{v^2}{c^2}}}##.

The following is a Lorentz boost in the ##y## direction:
$$\begin{bmatrix} \gamma & 0 & -\beta \gamma & 0 \\ 0 & 1 & 0 & 0 \\ -\beta \gamma & 0 & \gamma & 0 \\ 0 & 0 & 0 & 1\end{bmatrix}$$

However, despite knowing that this matrix of the Lorentz boost in the ##y## direction does not affect the ##x## and ##z## directions - rather only affects both time and the ##y## direction - I still don't quite get how people are able to get this calculation.

*Source: I learned this from StackExchange - https://physics.stackexchange.com/questions/30166/what-is-a-lorentz-boost-and-how-to-calculate-it

"Attempted Solution"
I heard from someone on StackExchange that once I "find the correct values for my boost of speed ##v## in the ##x## direction, I should do the same with the second boost in the ##y'## direction with a speed of ##w## - except that the matrix should have ##B, C,## and ##D## in the second row and/or column. Finally, I am supposed to multiply the matrics in one order, and then in the opposite order afterward".
What does everybody in the community think of such a solution process?
In addition, is the Lorentz boost in the ##x##-direction I have written above consist speed ##v## in the matrix like the question requires?
Beyond that, how should I go about finding the boost of speed ##w## in the ##y'## direction?

I apologize for the load of questions. But, perhaps not surprisingly, I am terribly confused by this problem and I sincerely hope to understand how to solve this question in an orderly fashion.

Regardless, I sincerely appreciate your help as well as sharing that thread with me. While I was able to learn a few points in the thread, I do admit a lot of its calculations escaped my understanding even though I tried to search on the web for assistance. I'll still continue to try to understand your thread, however, any help from you - or anybody else from the community - to push me in the right direction in solving the problem would be much appreciated. Thank you!
 

1. What is the Lorentz Transformation?

The Lorentz Transformation is a mathematical equation developed by Dutch physicist Hendrik Lorentz in the late 19th century to describe the relationship between space and time in Einstein's theory of special relativity. It explains how measurements of space and time change for an observer moving at a constant velocity relative to another observer.

2. What are boosts in special relativity?

Boosts are a type of transformation in special relativity that describe the change in coordinates between two reference frames in relative motion. They involve a velocity parameter and can be used to calculate how measurements of space and time change between the two frames.

3. How do boosts affect the perception of time and space?

Boosts in special relativity cause time and space to be perceived differently by observers in different reference frames. The perception of time slows down for an observer moving at a constant velocity relative to another observer, while the perception of space contracts in the direction of motion. These effects are known as time dilation and length contraction, respectively.

4. Can the Lorentz Transformation be applied to all velocities?

No, the Lorentz Transformation only applies to velocities that are constant and do not change direction. It cannot be used to calculate the effects of acceleration, which requires a more complex equation known as the Lorentz Transformation for accelerated frames.

5. What is the significance of the Lorentz Transformation in modern physics?

The Lorentz Transformation is an essential mathematical tool in modern physics, particularly in the field of special relativity. It allows scientists to accurately describe how measurements of space and time change for observers in different reference frames, and has been confirmed through numerous experiments and observations. It is also an important component of Einstein's famous equation, E=mc^2, which describes the relationship between mass and energy.

Similar threads

  • Introductory Physics Homework Help
Replies
4
Views
2K
  • Introductory Physics Homework Help
Replies
2
Views
908
  • Introductory Physics Homework Help
2
Replies
36
Views
821
  • Introductory Physics Homework Help
2
Replies
35
Views
3K
  • Introductory Physics Homework Help
Replies
2
Views
845
  • Introductory Physics Homework Help
Replies
8
Views
1K
  • Introductory Physics Homework Help
Replies
3
Views
373
  • Introductory Physics Homework Help
Replies
4
Views
987
  • Introductory Physics Homework Help
Replies
4
Views
1K
Replies
8
Views
1K
Back
Top